1
$\begingroup$

Two spherical bodies of mass $m_1$ and $m_2$ are placed in gravity free space as shown. Initially, the body with mass $m_2$ is at rest and the body with mass $m_1$ approaches the other body with a velocity $u \hat{i}$. An elastic oblique collision occurs. Find the ratio $\frac{m_1}{m_2}$ such that the maximum angle of deflection of $m_1$ is $60^\circ$.

I decided to solve this in the lab frame (or call it the ground frame), and I can't proceed until I know atleast one more parameter. If I solve it in the frame of the mass centre, as you will later see, it will be quite trivial. Can someone help m solve it in the ground frame?

Geogebra Diagram

My solution in the lab frame: Let $\angle{EAC} = \phi$, $\angle{JCI}= \beta$ and $\angle{FAE} = \alpha$ (see figure). In the collision, the impulse provided by contact force during collision is nearly zero and hence momentum can be conserved along the line of collision AC. Let $v_1$ be the speed if the body with mass $m_1$ after collision (at an angle of $\alpha$ with horizontal) and $v_2$ be speed of the body with mass $m_2$ after collision (at an angle of $\beta$ with the line of collision). Using conservation of momentum along line of collision, $$m_1 u \cos{\phi} = m_1 v_1 \cos{(\phi +\alpha)} + m_2 v_2 \cos{\beta}$$ Since the collision is elastic, $$e = 1 = \frac{v_2 \cos{\beta} - v_1 \cos{(\phi +\alpha)}}{u \cos{\phi} - 0}$$ Solving these two equations, we get $$\frac{(\frac{m_1}{m_2} +1)}{(\frac{m_1}{m_2} - 1)} \cdot \frac{v_1}{u} = \frac{\cos{\phi}}{\cos{(\phi + \alpha)}}$$ Let $\frac{m_1}{m_2} = \kappa$, now we use the fact that no motion is affected along the line perpendicular to the line of collision. So $\frac{v_1}{u} = \frac{\sin{\phi}}{\sin{(\phi+\alpha)}}$ and putting this in the expression we had got from the first two equations, we get $$\kappa = \frac{\tan{(\phi+\alpha)}+\tan{\phi}}{\tan{(\phi+\alpha)} - \tan{\phi}}$$ Now we need to find $\kappa$ such that the maximum angle of deflection of $m_1$ is $60^\circ$, which is equivalent to saying that $\tan{\alpha} \leq \sqrt{3}$. Just using the boundary condition of $\tan{\alpha} = \sqrt{3}$ should yield the result. This gives us the quadratic equation $$\kappa = 1+\frac{2}{\sqrt{3} (\cot{\phi} - \tan{\phi})}$$ which should be the desired answer.

But the problems with this result are 1) the maximum value of angle deflection is $\phi$ dependent, or dependent on the shape of the bodies, and 2) $\phi$ is an unknown and 3) it is not the correct answer. Is there some way in which we can calculate the numerical value of $\kappa$ in the ground frame, irrespective of any unknown parameters? And can someone point out the errors in this solution?

A hint for solving in the CM frame: Solving this problem in the frame of the centre of mass is quite beautiful. Here's a line of thought that'll lead to the answer: Use this well known fact- In elastic collisions, momentum vectors of the colliding bodies initially and finally are equal and opposite! (Try proving it as well, and try to think what'll happen in a general collision.) This would lead us to the conclusion that the speeds of the two bodies will remain same after collision in the frame of mass centre. So $\sin{\phi_{\text{max}}} = \frac{v_{\text{1, CM}}}{v_{\text{CM}}}$. Now it should be trivial from here. ;)

$\endgroup$
4
  • $\begingroup$ The information about the contact force is irrelevant, the contact force impulse is unimportant because the contact forces come in an opposing pair so that the change in momentum of the first ball is carried through as the change in momentum of the second ball. $\endgroup$
    – Triatticus
    Commented Jul 2, 2019 at 18:53
  • $\begingroup$ I know, but I never calculated the contact force impulse. Please read where I've written: 'But the problems with this result are...' $\endgroup$
    – kushal
    Commented Jul 3, 2019 at 2:11
  • $\begingroup$ You don't need to calculate it, it is unimportant in any step of this problem is what I'm saying and doesn't enter into the analysis at all. $\endgroup$
    – Triatticus
    Commented Jul 3, 2019 at 3:46
  • $\begingroup$ Yes, but I never took it into account at all. $\endgroup$
    – kushal
    Commented Jul 3, 2019 at 4:04

0

Your Answer

By clicking “Post Your Answer”, you agree to our terms of service and acknowledge you have read our privacy policy.